Đến nội dung

Hình ảnh

CMR: $\sum_{k=0}^{p-1} \left\lfloor\dfrac{k^2}{p}\right\rfloor = $ $\dfrac{(p-1)(p-2)}{3}$

- - - - - nguyenta98

  • Please log in to reply
Chủ đề này có 6 trả lời

#1
hxthanh

hxthanh

    Tín đồ $\sum$

  • Hiệp sỹ
  • 3921 Bài viết
Cho $p=4n+1$ là số nguyên tố.
Chứng minh rằng

$\sum_{k=0}^{p-1} \left\lfloor\dfrac{k^2}{p}\right\rfloor =\dfrac{(p-1)(p-2)}{3}$

#2
duongtoi

duongtoi

    Thiếu úy

  • Thành viên
  • 747 Bài viết
Em xem lại đề bài xem nhé.
$\sum_{k=0}^{p-1}\left [ \frac{k^2}{p} \right ]= \frac{\displaystyle\sum_{k=0}^{p-1}k^2}{p}=\frac{(p-1)p(2p-1)}{6p}=\frac{(p-1)(2p-1)}{6}$.

#3
hxthanh

hxthanh

    Tín đồ $\sum$

  • Hiệp sỹ
  • 3921 Bài viết

Em xem lại đề bài xem nhé.
$\sum_{k=0}^{p-1}\left [ \frac{k^2}{p} \right ]= \frac{\displaystyle\sum_{k=0}^{p-1}k^2}{p}=\frac{(p-1)p(2p-1)}{6p}=\frac{(p-1)(2p-1)}{6}$.

Đề bài không sai đâu anh ơi!
Cái $\left\lfloor x\right\rfloor$ là phần nguyên của $x$ đấy ạ!

#4
nguyenta98

nguyenta98

    Thượng úy

  • Hiệp sỹ
  • 1259 Bài viết

Cho $p=4n+1$ là số nguyên tố.
Chứng minh rằng

$\sum_{k=0}^{p-1} \left\lfloor\dfrac{k^2}{p}\right\rfloor =\dfrac{(p-1)(p-2)}{3}$

Giải như sau:
Trước tiên ta thấy $\left\lfloor\dfrac{i^2}{p}\right\rfloor=\dfrac{i^2-a_i}{p}$ với $a_i$ là số dư của $i^2$ khi chia cho $p$
Thật vậy đặt $i^2=pk+a_i \Rightarrow k=\dfrac{i^2-a_i}{p}$
Mà $\dfrac{i^2}{p}=k+\dfrac{a_i}{p} \Rightarrow \left\lfloor\dfrac{i^2}{p}\right\rfloor=\left\lfloor k+\dfrac{a_i}{p}\right\rfloor=k$ (do $0\le a_i<p$)
Như vậy nếu giả sử $1^2 \equiv a_1 \pmod{p}, 2^2 \equiv a_2 \pmod{p},...,(p-1)^2 \equiv a_{p-1} \pmod{p}$
Khi ấy $\sum_{k=0}^{p-1} \left\lfloor\dfrac{k^2}{p}\right\rfloor=\dfrac{1^2-a_1}{p}+\dfrac{2^2-a_2}{p}+...+\dfrac{(p-1)^2-a_{p-1}}{p}=\dfrac{1^2+2^2+...+(p-1)^2}{p}-\dfrac{a_1+a_2+a_3+...+a_{p-1}}{p}$
Ta cần cm $\dfrac{1^2+2^2+...+(p-1)^2}{p}-\dfrac{a_1+a_2+a_3+...+a_{p-1}}{p}=\dfrac{(p-1)(p-2)}{3}$
Ta thấy $\dfrac{1^2+2^2+...+(p-1)^2}{p}=\dfrac{(p-1)p(2p-1)}{6p}=\dfrac{(p-1)(2p-1)}{6}$
Như vậy cần cm $\dfrac{a_1+a_2+a_3+...+a_{p-1}}{p}=\dfrac{(p-1)(2p-1)}{6}-\dfrac{(p-1)(p-2)}{3}=\dfrac{p-1}{2}$
Hay cm $a_1+a_2+...+a_{p-1}=\dfrac{p-1}{2}.p$
Ta thấy $i^2 \equiv (p-i)^2 \pmod{p}$ với $1\le i<p$ vì tương đương $(p-i)^2-i^2=p^2-2pi \vdots p$ hiển nhiên đúng
Như thế suy ra $a_i \equiv a_{p-i} \pmod{p}$ và từ ấy suy ra $a_i=a_{p-1}$ do $a_i,a_{p-1}<p$
Như vậy ta chỉ cần chứng minh $a_1+a_2+...+a_{\frac{p-1}{2}}=\dfrac{p-1}{4}.p$
Giờ ta có một nhận định sau:
Xét tập $A=\{1,2,3,...,\frac{p-1}{2}\}$ khi ấy với mỗi phần tử $a \in A$ thì tồn tại duy nhất $b \in B$ với $b$ khác $a$ sao cho $a^2+b^2 \vdots p$
Thật vậy xét số $-a^2$
Ta cần cm tồn tại $b^2 \equiv -a^2 \pmod{p} \Rightarrow b^2 \equiv (-1).a^2 \pmod{p}$
Ta chọn $b \equiv ua \pmod{p}$ khi ấy $u^2 \equiv (-1) \pmod{p}$ khi ấy $(-1)$ là số chính phương $mod(p)$ nhưng đây là điều hiển nhiên vì theo định nghĩa scp $mod(p)$ thì $(-1)^{\frac{p-1}{2}} \equiv 1 \pmod{p}$ mà $p=4n+1$ nên nó đúng
Do đó tồn tại $u^2 \equiv (-1) \pmod{p}$ khi ấy cũng tồn tại $b$ sao cho $b \equiv ua \pmod{p}$ khi đó $b^2 \equiv -a^2 \pmod{p} \Rightarrow b^2+a^2 \vdots p$
Ta cm $b$ là duy nhất, thật vậy giả sử có $b'$ sao cho $b'^2+a^2 \vdots p$ khi ấy $b^2 \equiv b'^2 \pmod{p} \Rightarrow b'+b \vdots p$ hoặc $b-b' \vdots p$ nhưng đây là điều vô lí do $b'+b\le 2.\dfrac{p-1}{2}<p$ và $b-b'<p$
Do đó tồn tại duy nhất $b \in A$ sao cho $a^2+b^2 \vdots p$ khi ấy gọi $x,y$ là số dư của $a,b$ chia cho $p$ suy ra $x+y \vdots p$ mà $x<p,y<p \Rightarrow x+y<2p$ nên $x+y=p$
Trở lại bài toán với tổng $a_1+a_2+...+a_{\frac{p-1}{2}}$ theo nhận định trên với mỗi $a_i$ tồn tại duy nhất $a_j$ sao cho $a_i+a_j=p$
Nhóm thành cặp, do $p=4n+1$ nên $\dfrac{p-1}{4}$ nguyên
Suy ra $a_1+a_2+...+a_{\frac{p-1}{2}}=p.\dfrac{p-1}{4}$ đây là điều ta cần nên có $đpcm$
Bài toán được giải quyết.

P/S bài này tuy là phần nguyên nhưng theo mình dùng số học là nhiều :) dùng scp $mod(p)$ và hệ thặng dư (tuy mình không nói trong bài)

Bài viết đã được chỉnh sửa nội dung bởi nguyenta98: 13-11-2012 - 20:59


#5
hxthanh

hxthanh

    Tín đồ $\sum$

  • Hiệp sỹ
  • 3921 Bài viết
Chính xác là như vậy!

$\sum_{k=0}^{p-1} (k^2\mod p)=\sum_{k=0}^{p-1}\left(k^2-p\left\lfloor\dfrac{k^2}{p}\right\rfloor\right)=\dfrac{p(p-1)(2p-1)}{6}-pS$

Do đó: $S=\dfrac{(p-1)(2p-1)}{6}-\dfrac{1}{p}\sum_{k=0}^{p-1}(k^2\mod p)$

Phần tiếp theo không khác gì lời giải của nguyenta98

#6
hxthanh

hxthanh

    Tín đồ $\sum$

  • Hiệp sỹ
  • 3921 Bài viết
Với các số nguyên tố $p=4n+1$ thì ta chứng minh được $\sum_{k=0}^{p-1} (k^2\mod p)=\dfrac{p(p-1)}{2}$
Vậy còn các số nguyên tố $p'=4n+3$ thì sao các bạn nhỉ?

#7
hxthanh

hxthanh

    Tín đồ $\sum$

  • Hiệp sỹ
  • 3921 Bài viết
"Thu hẹp" bài này với $p=5$ còn đoạn lấy tổng là từ $\overline{1,n}$ ta có bài toán:

Chứng minh rằng: $\sum_{k=1}^n \left\lfloor\dfrac{1}{2}+\dfrac{k^2}{5}\right\rfloor = \left\lfloor\dfrac{1}{2}+\sum_{k=1}^n \dfrac{k^2}{5}\right\rfloor$





Được gắn nhãn với một hoặc nhiều trong số những từ khóa sau: nguyenta98

1 người đang xem chủ đề

0 thành viên, 1 khách, 0 thành viên ẩn danh